The dot plot shows the number of wins for 16 baseball teams. Which statement about thedata is true?.Baseball Team Wins•0123 4 5 6 7 8Number of WinsThere is a data point at 8, so most teams won 8 games.The data are clustered around 2, so most teams won exactly 2games.The data are clustered from 4 to 7, so most toams lost 4 to 7gamos.The data are clustered from 1 to 3, so most teams won 1 to 3games.

The Dot Plot Shows The Number Of Wins For 16 Baseball Teams. Which Statement About Thedata Is True?.Baseball

Answers

Answer 1

We can see on the graph that the dots represent a team, and on the x-axis is the number of wins. Looking at the graph we can see that a lot of teams won around 1-3 and just one team won 8 times, therefore, the correct answer is: The data are clustered from 1 to 3, so most teams won 1 to 3 games


Related Questions

KFind the future value and interest earned if $8806.54 is invested for 7 years at 4% compounded (a) semiannually and (b) continuously.(a) The future value when interest is compounded semiannually is approximately $ 11,620.04.(Type an integer or decimal rounded to the nearest hundredth as needed.)The interest earned is approximately $ 2813.5.(Type an integer or decimal rounded to the nearest hundredth as needed.)(b) The future value when interest is compounded continuously is approximately $(Type an integer or decimal rounded to the nearest hundredth as needed.)

Answers

Given:

The principal amount = $8806.54

Rate of interest = 4%

Time = 7 years

Required:

Find the future value when interest is compounded continuously.

Explanation:

The future value is calculated by using the formula:

[tex]Future\text{ value = Ae}^{rt}[/tex]

Where A = amount

r = rate of interest

t = time period

Substitute the given values in the formula:

[tex]\begin{gathered} Future\text{ value = 8806.54\lparen e}^{0.04\times7}) \\ =8806.54(e^{0.28}) \\ =8806.54\times1.323 \\ =11,651.0524 \\ \approx11,651.05 \end{gathered}[/tex]

Interest = 11,651.05 - 8806.54

= 2844.51

Final Answer:

The future value when interest is compounded continuously is approximately $11,651.05.

The earned interest is approximately $2844.51

Solve each equation by using the square root property. 2x^2–9=11

Answers

We want to solve

2x^2–9=11

First, isolate the portion of the equation that's actually being squared. That is:

2x^2 = 11 + 9

that is equivalent to:

2x^2 = 20

that is equivalent to

x^2 = 20/ 2 = 10

that is

x^2 = 10

Now square root both sides and simplify, that is:

[tex]\sqrt[]{x^2\text{ }}=\text{ }\sqrt[]{10}[/tex]

we know that the square root is the inverse function of the function x^ 2, so we can cancel the square :

[tex]x\text{ = }\sqrt[]{10}[/tex]

but note that there is always the possibility of two roots for every square root: one positive and one negative: so the final answer is:

[tex]x\text{ = +/- }\sqrt[]{10}[/tex]

Zero and negative exponentswrite in simplest for without zero or negative exponents10c -²

Answers

[tex]\frac{10}{c^2}[/tex]

Explanation

remember some properties of the exponents

[tex]\begin{gathered} a^m\cdot a^n=a^{m+n} \\ (a^m)^n=a^{m\cdot n} \\ a^{-m}=\frac{1}{a^m} \end{gathered}[/tex]

Hence,apply

[tex]10c^{-2}=10\cdot c^{-2}=\frac{10}{c^2}[/tex]

I hope this helps you

I need help solving this logarithmic equation. I need answered step by step,

Answers

Okay, here we have this:

We need to solve the following equation for n:

[tex]\log _8n=3[/tex]

To solve this equation we will pass the logarithm to its exponential form:

[tex]\begin{gathered} n=8^3 \\ n=8\cdot8\cdot8 \\ n=512 \end{gathered}[/tex]

Finally we obtain that n=512.

Answer:

n = 512

Step-by-step explanation:

Solving logarithmic equations:

   Write logarithmic equations to exponential equation.

     [tex]\sf \log_8 \ n = 3\\\\\\ 8^3 = n\\\\[/tex]

    n = 8 * 8 *8

    n = 512

A bottler of drinking water fills plastic bottles with a mean volume of 993 milliliters (mL) and standard deviation of 7 mL. The fill volumes are normally distributed. What proportion of bottles have volumes between 988 mL and 991 mL?

Answers

Given data:

Mean: 993mL

Standard deviation: 7mL

Find p(988

1. Find the z-value corresponding to (x>988), use the next formula:

[tex]\begin{gathered} z=\frac{x-\mu}{\sigma} \\ \\ z=\frac{988-993}{7}=-0.71 \end{gathered}[/tex]

2. Find the z-value corresponding to (x<991):

[tex]z=\frac{991-993}{7}=-0.29[/tex]

3. Use a z score table to find the corresponding values for the z-scores above:

For z=-0.71: 0.2389

For x=-0.29: 0.3859

4. Subtract the lower limit value (0.2389) from the upper limit value (0.3859):

[tex]0.3859-0.2389=0.147[/tex]

5. Multiply by 100 to get the percentage:

[tex]0.147*100=14.7[/tex]Then, 14.7% of the bottles have volumes between 988mL and 991mL

Step 1 Step 2 Step 3 Using the figures above, how many small squares will there be in step 4 and step 15? a. Step 4 = b. Step 15 =

Answers

Step 4 = 16 squares

Step 15 = 225 squares

1) In the 1st step we can see, 1 square. In the 2nd, 4, and on the third one 9

So there's a sequence, 1, 4, 9

2) We can write the positions and raise them to the 2nd power we can see how it grows:

position (steps) n | 1 | 2 | 3

# squares | 1 | 4 | 9

3) We can derive a formula for that sequence:

[tex]a_n=n^2[/tex]

Following this rule, we can find that

Step 4 = 4² = 16 squares

Step 15 = 15² = 225 squares

A small publishing company is planning to publish a new book. the production cost will include one-time fix costs (such as editing) and variable costs (such as printing). There are two production methods it could use. With one method, the one-timed fixed costs will total $15,756, and the variable costs will be $23.50 per book. With the other method, the one-timed costs will total $48,108, and the variable costs will be $12 per book. For how many books produced will the costs from the two methods be the same?

Answers

What we must do is equal both equations like this:

[tex]15756+23.5\cdot x=48108+12\cdot x[/tex]

solving for x (numbers of books):

[tex]\begin{gathered} 23.5\cdot x-12\cdot x=48108-15756 \\ 11.5\cdot x=32352 \\ x=\frac{32352}{11.5} \\ x=2813.2=2813 \end{gathered}[/tex]

In aproximately 2813 books

In triangle ABC, AB12, BC18, and m B = 75°. What are the approximate length of AC and measure of A

Answers

Length AB = 12cm

BC = 18cm

mB = 75^

true or false the diameter is equal to twice the radius

Answers

True, the diameter = twice the radius

5: =3:21 its equivalent ratios

Answers

The number that makes the ratios equivalent is 35. Thus, the ratio becomes 5:35 = 3:21

Equivalent ratios

From the question, we are to determine the number that will make the two ratios equivalent ratios.

From the given equation,

5: = 3:21

Let the unknown number be x.

Thus,

The equation becomes

5:x = 3:21

Then,

We can write that

5/x = 3/21

Cross multiply

x × 3 = 5 × 21

3x = 105

Divide both sides by 3

3x/3 = 105/3

x = 35

Hence, the number is 35

Learn more on Equivalent ratios here: https://brainly.com/question/2328454

#SPJ1

Mrs. Cavazos car traveled 192 miles on 6 gallons of gas. Find the unit rate per gallon

Answers

To find the unit rate per gallon, we are going to divide 192 by 6

[tex]\frac{192}{6}=32[/tex]

The car gets 32 miles per gallon.

Martina used a total of 4 3/4 gallons of gas while driving her car. Each hour she was driving, she used 5/6 gallons of gas. What was the total number of hours she was driving?

Answers

The number of hours she was driving = 5.7 hours or in fraction 57/10 hours.

What is fraction?

A fraction is a number that represents a part of a whole.

Generally, the fraction can be a portion of any quantity out of the whole thing and the whole can be any specific things or value.

Given, a total gallons is in mixed fraction 4 3/4

can be written as

16+3/4 = 19/4

Let x be the hours she was driving.

The she used 5/6 gallons.

x (5/6) = 19/4

x = 19/4(6/5)

x = 5.7 hours

To know more about fraction, visit:

https://brainly.com/question/26386522

#SPJ1

what is the median 14,6,-11,-6,5,10

Answers

The median of a set of values is the values that divide the set into two subsets, one containing all the values less than the median, and another containing all the values greater than the median.

So, to find the median, let's first rewrite the given values in ascending order:

-11, -6, 5, 6, 10, 14

If the set had an odd number of values, the value in the middle, after rewriting them as we did, would be the median.

Nevertheless, the number of values in this set is even. When this happens, the median corresponds to the mean of the two central numbers.

In this case, the two central numbers are 5 and 6. Their mean is:

(5 + 6)/2 = 11/2 = 5.5

Thus, the median is 5.5.

. Connect to Everyday Life In which situation is
a rounded number appropriate? Explain.
The number of
birds in a flock
The number of players on a
football field during a game

Answers

The situations that a rounded number is appropriate is both the given situations.

The number of birds in a flock.

The number of players on a football field during a game.

Both the give situation is correct.

What is an expression?

An expression is a way of writing a statement with more than two variables or numbers with operations such as addition, subtraction, multiplication, and division.

Example: 2 + 3x + 4y = 7 is an expression.

We have,

The number of birds in a flock.

This will always be a rounded number.

We never say that there are 3.3 birds in a flock

We always say that there are 33 birds in the flock.

The number of players on a football field during a game.

This is always a rounded number.

We never say that there are 3 and a half players or 4.5 players on a football field.

We always say 24 players on a football field.

Thus,

The situations that a rounded number is appropriate is both the given situations.

The number of birds in a flock.

The number of players on a football field during a game.

Both the given situation is correct.

Learn more about expressions here:

https://brainly.com/question/3118662

#SPJ1

Answer:

The situations that a rounded number is appropriate is both the given situations.The number of birds in a flock.The number of players on a football field during a game.Both the give situation is correct.What is an expression?An expression is a way of writing a statement with more than two variablesor numbers with operations such as addition, subtraction, multiplication, and division.Example: 2 + 3x + 4y = 7 is an expression.We have,The number of birds in a flock.This will always be a rounded number.We never say that there are 3.3 birds in a flockWe always say that there are 33 birds in the flock.The number of players on a football field during a game.This is always a rounded number.We never say that there are 3 and a half players or 4.5 players on a football field.We always say 24 players on a football field.

Can you please help me find the area? Thank you. :)))

Answers

The figure shown in the picture is a rectangular shape that is missing a triangular piece. To determine the area of the figure you have to determine the area of the rectangle and the area of the triangular piece, then you have to subtract the area of the triangle from the area of the rectangle.

The rectangular shape has a width of 12 inches and a length of 20 inches. The area of the rectangle is equal to the multiplication of the width (w) and the length (l), following the formula:

[tex]A=w\cdot l[/tex]

For our rectangle w=12 in and l=20 in, the area is:

[tex]\begin{gathered} A_{\text{rectangle}}=12\cdot20 \\ A_{\text{rectangle}}=240in^2 \end{gathered}[/tex]

The triangular piece has a height of 6in and its base has a length unknown. Before calculating the area of the triangle, you have to determine the length of the base, which I marked with an "x" in the sketch above.

The length of the rectangle is 20 inches, the triangular piece divides this length into three segments, two of which measure 8 inches and the third one is of unknown length.

You can determine the value of x as follows:

[tex]\begin{gathered} 20=8+8+x \\ 20=16+x \\ 20-16=x \\ 4=x \end{gathered}[/tex]

x=4 in → this means that the base of the triangle is 4in long.

The area of the triangle is equal to half the product of the base by the height, following the formula:

[tex]A=\frac{b\cdot h}{2}[/tex]

For our triangle, the base is b=4in and the height is h=6in, then the area is:

[tex]\begin{gathered} A_{\text{triangle}}=\frac{4\cdot6}{2} \\ A_{\text{triangle}}=\frac{24}{2} \\ A_{\text{triangle}}=12in^2 \end{gathered}[/tex]

Finally, to determine the area of the shape you have to subtract the area of the triangle from the area of the rectangle:

[tex]\begin{gathered} A_{\text{total}}=A_{\text{rectangle}}-A_{\text{triangle}} \\ A_{\text{total}}=240-12 \\ A_{\text{total}}=228in^2 \end{gathered}[/tex]

The area of the figure is 228in²

Third-degree, with zeros of 2-i, 2+i and 3 and a leading coefficient of -4

Answers

Answer:

Step-by-step explanation:

A spinner is shown below. what is the probability that a 5 is spun?

Answers

Answer:

The probability that 5 is spun is;

[tex]\begin{gathered} P(5)=\frac{2}{9} \\ or \\ P(5)=22.22\text{\%} \end{gathered}[/tex]

Explanation:

Given the figure in the attached image.

We will assume that each of the sectors are of the same size.

The probability of spinning a 5 is equal to the number of sectors with 5 divided by the total number of sectors.

[tex]\begin{gathered} n(5)=2 \\ n(\text{total)}=9 \end{gathered}[/tex]

So, the probability that 5 is spun is;

[tex]\begin{gathered} P(5)=\frac{n(5)}{n(\text{total)}}=\frac{2}{9} \\ P(5)=\frac{2}{9} \\ \text{ in percentage;} \\ P(5)=\frac{2}{9}\times100\text{\%} \\ P(5)=22.22\text{\%} \end{gathered}[/tex]

Therefore, the probability that 5 is spun is;

[tex]\begin{gathered} P(5)=\frac{2}{9} \\ or \\ P(5)=22.22\text{\%} \end{gathered}[/tex]

Simplify: 6-(-9) divided by -9/-4

Answers

Answer:

6 2/3

Explanation:

Given the expression:

[tex]\lbrack6-\mleft(-9\mright)\rbrack\div\frac{-9}{-4}[/tex]

First, we simplify to obtain:

[tex]=\lbrack6+9\rbrack\div-\frac{9}{-4}[/tex]

Note that -9/-4=9/4. The minus sign cancels each other out.

This gives us:

[tex]15\div\frac{9}{4}[/tex]

We then change the division sign to multiplication as shown below:

[tex]\begin{gathered} =15\times\frac{4}{9} \\ =\frac{60}{9} \\ =6\frac{6}{9} \\ =6\frac{2}{3} \end{gathered}[/tex]

First blank transitive propertySubtraction property of equalitySegment additionSubstitution property of equalitysecond blank AB does not equal YZ AC does not equal XZ AB equals YZ AC equals XZ

Answers

Given that:

[tex]BC=XY[/tex][tex]AB+BC\ne YZ+XY[/tex]

According to the Segment Addition if B lies between A and C, then:

[tex]AB+BC=AC[/tex]

In this case, knowing that:

[tex]AB+BC\ne YZ+XY[/tex]

And knowing that B lies between A and C, and Y lies between X and Z:

[tex]\begin{gathered} AB+BC=AC \\ YX+XY=XZ \end{gathered}[/tex]

Therefore, you can determine that:

[tex]AC\ne XZ[/tex]

Hence, the answers are:

- First blank: Third option (Segment addition).

- Second blank: Second option (AC does not equal XZ).

The two triangles below are similar. Also, m A = 15° and m ZC - 35° as shown below Find m2P, m 2Q, and m ZR. Assume the triangles are accurately drawn

Answers

Answer

Angle R = 15°

Angle P = 130°

Angle Q = 35°

Explanation

First noting that the sum of angles in a triangle is 180°.

We first need to calculate the Angle B for the first triangle.

Angle A + Angle B + Angle C = 180°

15° + Angle B + 35° = 180°

Angle B + 50° = 180°

Angle B = 180° - 50°

Angle B = 130°

We are then told to find Angles P, Q and R.

We are told that the two triangles are similar .

Two similar triangles will have the same angle measures.

So, we just need to note the corresponding angles and equate the unknowns.

Triangle ABC is similar to Triangle RPQ

Angle R = Angle A = 15°

Angle P = Angle B = 130°

Angle Q = Angle C = 35°

Hope this Helps!!!

question will be in picture

Answers

f(x) = -5x + 4

What is the value of x when f(x) = 29

To find x, equate -5x + 4 to 29.

-5x + 4 = 29

Next, collect like terms.

-5x = 29 - 4

-5x = 25

Finally divide through by -5 to find the value of x.

[tex]\begin{gathered} \frac{-5x}{-5}\text{ = }\frac{25}{-5} \\ x\text{ = -5} \end{gathered}[/tex]

Final answer

x = -5 Option C

For any right triangle, the side lengths of the triangle can be put in the equation a^2+ b^2 = c^2 where a, b, and c are the side lengths. A triangle with the side lengths 3 inches, 4 inches, and 5 inches is a right triangle. Which way(s) can you substitute the values into the equation to make it true? Which variable has to match the longest side length? Why?

Answers

It is given that the side lengths of any right triangle can be put in the equation:

[tex]a^2+b^2=c^2[/tex]

For a triangle with the side lengths 3 inches, 4 inches, and 5 inches, it can be substituted in two ways that will make the equation true:

Let a=3, b=4, and c=5:

[tex]\begin{gathered} 3^2+4^2=5^2 \\ \Rightarrow9+16=25 \\ \Rightarrow25=25 \end{gathered}[/tex]

Hence, the equation is true.

You can also substitute a=4, b=3, and c=5.

This will also give the same result.

Notice that variable c has to match the longest side length.

The reason for this is that equality can only hold if the longest side is the variable at the right, if not there'll be an inequality instead.

fill in the blank with the correct answer. the number _______is divisibel by 2, 3, 4, 5, and 6

a)44
b)180
c)280
d)385

Answers

Answer:

b) 180

Explanation:

[tex]180 / 2 = 90\\180 / 3 = 60\\180 / 4 = 45\\180 / 5 = 36\\180 / 6 = 30[/tex]

Hope you have a nice day and a nice Thanksgiving!

A brainiliest would also be nice. thx.

Select all of the true statements about to figure, if a scale factor is 2.

Answers

Given: The scale factor is 2 for the given figures

To Determine: The truth statements from the given options

The transformation shown is an enlargement. This means that each of the length of the pre-image multiplied by 2 would give the length of the image

This means

[tex]\begin{gathered} A^{\prime}B^{\prime}=2AB \\ A^{\prime}C^{\prime}=2AC \\ B^{\prime}C^{\prime}=2BC \end{gathered}[/tex]

For similar shapes, the angles are congruent and the sides are in proportion of the scale factor

Hence, the following are true statements of the given diagrams

A'C' = 2 AC, OPTION B

If AB = 6, then A'B' = 12, OPTION E

a vector s has the initial point (-2,-4) and terminal point (-1,3) write s in the form s = ai + bj

Answers

To write the vector s in the form s=ai + bj, we can use the next formula:

[tex]\vec{s}=(x_2-x_1)\vec{i}+(y_2-y_1)\vec{j}[/tex]

Where (x1,y1) are the coordinates of the initial point and (x2,y2) are the coordinates of the terminal point, by replacing these values we have:

[tex]\begin{gathered} \vec{s}=((-1)-(-2))\vec{i}+(3-(-4))\vec{j} \\ \vec{s}=((-1)+2)\vec{i}+(3+4)\vec{j} \\ \vec{s}=(1)\vec{i}+(7)\vec{j} \end{gathered}[/tex]

Then the vector s in the form s=ai+bj is: s= 1i + 7j

Find each unknown function value or x value for f(x) = 4x - 7 and g(x) = -3x + 5

Answers

Step 1

Find f(2)

[tex]To\text{ do this we substitute for f= 2 in f(x)}[/tex][tex]\begin{gathered} f(x)\text{ = }4x-7 \\ f(2)\text{ = 4(2) -7 = 8 - 7 = 1} \end{gathered}[/tex]

Step 2

Find f(0)

[tex]f(0)\text{ = 4(0) -7 = 0 - 7 = -7}[/tex]

Step 3

Find f(-3)

[tex]f(-3)\text{ = 4(-3) -7 = -12 -7 = -19}[/tex]

Step 4

Find x, when f(x) = -3

[tex]\begin{gathered} f(x)\text{ = -3}--------------(1) \\ f(x)\text{ =4x-7}---------------(2) \\ \text{Equate both equations} \\ -3=4x-7 \\ -3+7\text{ = 4x} \\ 4x\text{ = 4} \\ x\text{ = }\frac{4}{4}=1 \end{gathered}[/tex]

Jada bought an art kit with 50 colored pencils. She and her 3 sisters will share the pencils equally. How many pencils will each person receive? Will there be any pencils left over? If so, how many?

Answers

Each will get 16 coloured pencils and 2 will be the left over

Step-by-step explanation:

Give 10 pencil each then add 6 more for each one and the answer will be 16 each and multiple 3×16 =48 and remainder 2

Solve x2 + 5x = 0.Step 1. Factor x2 + 5x as the product of two linear expressions.

Answers

[tex]x^2+5x=0[/tex]

Taking common factor x:

[tex]x(x+5)=0[/tex]

Equal each factor to zero, and solve for x:

[tex]x=0[/tex][tex]\begin{gathered} x+5=0 \\ x=-5 \end{gathered}[/tex]

So, the solution is:

[tex]\begin{gathered} x_1=0 \\ x_2=-5 \end{gathered}[/tex]

The formula S=C(1+r) models inflation, where C = the value today, r = the annual inflation rate, and S = the inflated value t years from now. a. If the inflation rate is 6%, how much will a house now worth $465,000 be worth in 10 years? b. If the inflation rate is 3%, how much will a house now worth $510,000 be worth in 5 years?

Answers

The formula that models inflation is

[tex]S=C(1+r)^t[/tex]

C= value today

r= annual inflation rate → usually this value is given as a percentage, but when you input the value in the formula, you have to express it as a decimal value.

S= the inflated value given a determined period of time (t).

a.

r=6%=6/100=0.06/year

C=$465000

t=10 years

[tex]\begin{gathered} S=465000(1+0.06)^{10} \\ S=832744.1789 \end{gathered}[/tex]

The price of the house in 10 years at an inflation rate of 6% will be S=$832744.18

b.

r=3%=3/100=0.03/year

C=$510000

t=5years

[tex]\begin{gathered} S=510000(1+0.03)^5 \\ S=437954.3531 \end{gathered}[/tex]

The price of the house in 5 years at an inflation rate of 3% will be S=$437954.35

Find the image of the given pointunder the given translation.

Answers

Answer: P' = (4, 4)

Explanation

As the given point is (8, –3), then the transformation is:

[tex]T(x,y)=(x-4,y+7)=(8-4,-3+7)[/tex][tex]T(x,y)=(4,4)[/tex]

Other Questions
Im not a picky eater; plants or animals are just fine for me! What am I? The answer that I need starts with an H? Im confused Entrance Ticket on Translations Sarah graphed a triangle with vertices X (3,3) Y (4,1) Z (1,1). She asked her classmate Paul to translate the triangle (x-4) (y+2). Paul stated that the triangle will move down by 4 and right by 2 putting the triangle in the 4th quadrant. Graph the translation to see if Paul is correct? Explain your reasoning. Your explanation should include: What is a translation? Is Paul correct? Why or why not Which direction should you move the triangle? Which quadrant is the translation located? I need help to solve part B and give my reasoning in two collum proof. What is the total paymentrequired to pay off a promissorynote issued for $700.00 at 12%ordinary interest and a 180-dayterm?A. $760.00B. $742.00C. $712.00D. $721.60 Hello, I'm stuck on this.Question: Calculate the slant height of this come, identified by letter X. Give your answer to the nearest whole number. Two integers, c and d, have a product of -6. What is the greatest possible sum of cand d? High school band perform a concert on four different days bandsaw tickets each day as a fun raiser below the table shows the number of tickets sold in the amount collection of from sales slove with tablesHELPPP PLEASSEEEEEE 17. Is the following statement true or false? A triangle can have two angles that measure 100.Explain. i need help with question 2 Area of Triangles What is the area of this triangle? bh A 0 24 in? 8 in O 30 in 48 in 96 in valley view chicken coop's costs went from $0.05 per egg when they harvested from 50 chickens to $0.08 per egg when they harvested from 80 chickens. which of the following describes this scenario? a.) economies of scale b.) both diseconomies and economies of scale c.) constant scale d.) diseconomies of scale Four different stores have the same digital camera on sale. The original price and discounts offered by each store are listed below. Rank the stores from the cheapest to most expensive sale price of the camera.Store A: price $99.99 and discount of 15%Store B: price $95.99 and discount of 12%Store C: price $90.99 and discount of 10%Store D: price $89.99 and successive discounts of 5% and 5% The triangle has the sides AB = 4cm BC = 6cm AC = 8cmCalculate the triangles highest corner Use the data in the following table, which shows the results of a survey of 2000 gamers concerning their favorite home video game systems, organized by age group. If a survey participant is selected at random, determine the probability of the following. Round to the nearest hundredth.The participant prefers the Nintendo Wii U system. Can you help me with this, Ive already completed the assignment but trying to go back and figure out what I missed and what I did wrong Deon had $25 then he spent $15 on lunch. What percentage of his money did deon spend on lunch The list price for a case of medicine is $275.49 Your pharmacy will receive a 18% trade discount. What is the amount of the discount? b) What is the net cost of the case of medicine? The Americans who wrote the Declaration of Independence wanted support from people in the colonies and from other countries. Which country did the colonists ask for help during the revolution? A. France B. Russia C. Germany D. Spain Whats the other side?Image attachedplease show work Sarah is going to invest $79,000 and leave it in an account for 7 years. Assuming theinterest is compounded quarterly, what interest rate, to the nearest tenth of apercent, would be required in order for Sarah to end up with $121,000?